$H$ is a subgroup of a finite group $G$ such that $|H|$ and $big([G:H]-1big)!$ are relatively prime. Prove...












3












$begingroup$



Suppose that $H$ is a subgroup of a finite group $G$ and that $|H|$ and $big([G:H]-1big)!$ are relatively prime. Prove that $H$ is normal in G




Let $[G:H]=m$



Let $G$ act on set $A$ of left cosets of $H$ in $G$ under left multiplication, then we have



$$phi : G to S_Asimeq S_m$$



where $$K= kerphi subseteq H$$
$$frac{G}{K} simeq phi(G)leq S_m$$



So $$[G:K] mid m!$$



$$Rightarrow [G:H][H:K]mid m!$$



$$Rightarrow [H:K]|(m-1)!$$



But $gcdbig(|H|,(m-1)!big) = 1 Rightarrow |K|>1 $



How do I proceed to prove $K=H lhd G$?



Or is this approach wrong










share|cite|improve this question











$endgroup$












  • $begingroup$
    This is a more general version of the fact that a subgroup of index $2$ is always normal
    $endgroup$
    – So Lo
    Dec 11 '18 at 15:33










  • $begingroup$
    Instead you can note that the condition implies that any prime divisor in $|H|$ is greater than the index of $H$. Then if $K$ is any subgroup of the same order as $H$ you can see from the possible order that $HK = H$.
    $endgroup$
    – Tobias Kildetoft
    Dec 11 '18 at 15:39
















3












$begingroup$



Suppose that $H$ is a subgroup of a finite group $G$ and that $|H|$ and $big([G:H]-1big)!$ are relatively prime. Prove that $H$ is normal in G




Let $[G:H]=m$



Let $G$ act on set $A$ of left cosets of $H$ in $G$ under left multiplication, then we have



$$phi : G to S_Asimeq S_m$$



where $$K= kerphi subseteq H$$
$$frac{G}{K} simeq phi(G)leq S_m$$



So $$[G:K] mid m!$$



$$Rightarrow [G:H][H:K]mid m!$$



$$Rightarrow [H:K]|(m-1)!$$



But $gcdbig(|H|,(m-1)!big) = 1 Rightarrow |K|>1 $



How do I proceed to prove $K=H lhd G$?



Or is this approach wrong










share|cite|improve this question











$endgroup$












  • $begingroup$
    This is a more general version of the fact that a subgroup of index $2$ is always normal
    $endgroup$
    – So Lo
    Dec 11 '18 at 15:33










  • $begingroup$
    Instead you can note that the condition implies that any prime divisor in $|H|$ is greater than the index of $H$. Then if $K$ is any subgroup of the same order as $H$ you can see from the possible order that $HK = H$.
    $endgroup$
    – Tobias Kildetoft
    Dec 11 '18 at 15:39














3












3








3


2



$begingroup$



Suppose that $H$ is a subgroup of a finite group $G$ and that $|H|$ and $big([G:H]-1big)!$ are relatively prime. Prove that $H$ is normal in G




Let $[G:H]=m$



Let $G$ act on set $A$ of left cosets of $H$ in $G$ under left multiplication, then we have



$$phi : G to S_Asimeq S_m$$



where $$K= kerphi subseteq H$$
$$frac{G}{K} simeq phi(G)leq S_m$$



So $$[G:K] mid m!$$



$$Rightarrow [G:H][H:K]mid m!$$



$$Rightarrow [H:K]|(m-1)!$$



But $gcdbig(|H|,(m-1)!big) = 1 Rightarrow |K|>1 $



How do I proceed to prove $K=H lhd G$?



Or is this approach wrong










share|cite|improve this question











$endgroup$





Suppose that $H$ is a subgroup of a finite group $G$ and that $|H|$ and $big([G:H]-1big)!$ are relatively prime. Prove that $H$ is normal in G




Let $[G:H]=m$



Let $G$ act on set $A$ of left cosets of $H$ in $G$ under left multiplication, then we have



$$phi : G to S_Asimeq S_m$$



where $$K= kerphi subseteq H$$
$$frac{G}{K} simeq phi(G)leq S_m$$



So $$[G:K] mid m!$$



$$Rightarrow [G:H][H:K]mid m!$$



$$Rightarrow [H:K]|(m-1)!$$



But $gcdbig(|H|,(m-1)!big) = 1 Rightarrow |K|>1 $



How do I proceed to prove $K=H lhd G$?



Or is this approach wrong







abstract-algebra group-theory group-actions normal-subgroups quotient-group






share|cite|improve this question















share|cite|improve this question













share|cite|improve this question




share|cite|improve this question








edited Dec 15 '18 at 21:45









Batominovski

1




1










asked Dec 11 '18 at 15:17









So LoSo Lo

62729




62729












  • $begingroup$
    This is a more general version of the fact that a subgroup of index $2$ is always normal
    $endgroup$
    – So Lo
    Dec 11 '18 at 15:33










  • $begingroup$
    Instead you can note that the condition implies that any prime divisor in $|H|$ is greater than the index of $H$. Then if $K$ is any subgroup of the same order as $H$ you can see from the possible order that $HK = H$.
    $endgroup$
    – Tobias Kildetoft
    Dec 11 '18 at 15:39


















  • $begingroup$
    This is a more general version of the fact that a subgroup of index $2$ is always normal
    $endgroup$
    – So Lo
    Dec 11 '18 at 15:33










  • $begingroup$
    Instead you can note that the condition implies that any prime divisor in $|H|$ is greater than the index of $H$. Then if $K$ is any subgroup of the same order as $H$ you can see from the possible order that $HK = H$.
    $endgroup$
    – Tobias Kildetoft
    Dec 11 '18 at 15:39
















$begingroup$
This is a more general version of the fact that a subgroup of index $2$ is always normal
$endgroup$
– So Lo
Dec 11 '18 at 15:33




$begingroup$
This is a more general version of the fact that a subgroup of index $2$ is always normal
$endgroup$
– So Lo
Dec 11 '18 at 15:33












$begingroup$
Instead you can note that the condition implies that any prime divisor in $|H|$ is greater than the index of $H$. Then if $K$ is any subgroup of the same order as $H$ you can see from the possible order that $HK = H$.
$endgroup$
– Tobias Kildetoft
Dec 11 '18 at 15:39




$begingroup$
Instead you can note that the condition implies that any prime divisor in $|H|$ is greater than the index of $H$. Then if $K$ is any subgroup of the same order as $H$ you can see from the possible order that $HK = H$.
$endgroup$
– Tobias Kildetoft
Dec 11 '18 at 15:39










1 Answer
1






active

oldest

votes


















4












$begingroup$

Note that $[H:K]$ divides both $|H|$ and $big([G:H]-1big)!=(m-1)!$. Therefore, it divides the greatest common divisor of $|H|$ and $big([G:H]-1big)!$, which is $1$. Hence, $[H:K]=1$, making $K=H$.






share|cite|improve this answer









$endgroup$













    Your Answer





    StackExchange.ifUsing("editor", function () {
    return StackExchange.using("mathjaxEditing", function () {
    StackExchange.MarkdownEditor.creationCallbacks.add(function (editor, postfix) {
    StackExchange.mathjaxEditing.prepareWmdForMathJax(editor, postfix, [["$", "$"], ["\\(","\\)"]]);
    });
    });
    }, "mathjax-editing");

    StackExchange.ready(function() {
    var channelOptions = {
    tags: "".split(" "),
    id: "69"
    };
    initTagRenderer("".split(" "), "".split(" "), channelOptions);

    StackExchange.using("externalEditor", function() {
    // Have to fire editor after snippets, if snippets enabled
    if (StackExchange.settings.snippets.snippetsEnabled) {
    StackExchange.using("snippets", function() {
    createEditor();
    });
    }
    else {
    createEditor();
    }
    });

    function createEditor() {
    StackExchange.prepareEditor({
    heartbeatType: 'answer',
    autoActivateHeartbeat: false,
    convertImagesToLinks: true,
    noModals: true,
    showLowRepImageUploadWarning: true,
    reputationToPostImages: 10,
    bindNavPrevention: true,
    postfix: "",
    imageUploader: {
    brandingHtml: "Powered by u003ca class="icon-imgur-white" href="https://imgur.com/"u003eu003c/au003e",
    contentPolicyHtml: "User contributions licensed under u003ca href="https://creativecommons.org/licenses/by-sa/3.0/"u003ecc by-sa 3.0 with attribution requiredu003c/au003e u003ca href="https://stackoverflow.com/legal/content-policy"u003e(content policy)u003c/au003e",
    allowUrls: true
    },
    noCode: true, onDemand: true,
    discardSelector: ".discard-answer"
    ,immediatelyShowMarkdownHelp:true
    });


    }
    });














    draft saved

    draft discarded


















    StackExchange.ready(
    function () {
    StackExchange.openid.initPostLogin('.new-post-login', 'https%3a%2f%2fmath.stackexchange.com%2fquestions%2f3035390%2fh-is-a-subgroup-of-a-finite-group-g-such-that-h-and-biggh-1-big%23new-answer', 'question_page');
    }
    );

    Post as a guest















    Required, but never shown

























    1 Answer
    1






    active

    oldest

    votes








    1 Answer
    1






    active

    oldest

    votes









    active

    oldest

    votes






    active

    oldest

    votes









    4












    $begingroup$

    Note that $[H:K]$ divides both $|H|$ and $big([G:H]-1big)!=(m-1)!$. Therefore, it divides the greatest common divisor of $|H|$ and $big([G:H]-1big)!$, which is $1$. Hence, $[H:K]=1$, making $K=H$.






    share|cite|improve this answer









    $endgroup$


















      4












      $begingroup$

      Note that $[H:K]$ divides both $|H|$ and $big([G:H]-1big)!=(m-1)!$. Therefore, it divides the greatest common divisor of $|H|$ and $big([G:H]-1big)!$, which is $1$. Hence, $[H:K]=1$, making $K=H$.






      share|cite|improve this answer









      $endgroup$
















        4












        4








        4





        $begingroup$

        Note that $[H:K]$ divides both $|H|$ and $big([G:H]-1big)!=(m-1)!$. Therefore, it divides the greatest common divisor of $|H|$ and $big([G:H]-1big)!$, which is $1$. Hence, $[H:K]=1$, making $K=H$.






        share|cite|improve this answer









        $endgroup$



        Note that $[H:K]$ divides both $|H|$ and $big([G:H]-1big)!=(m-1)!$. Therefore, it divides the greatest common divisor of $|H|$ and $big([G:H]-1big)!$, which is $1$. Hence, $[H:K]=1$, making $K=H$.







        share|cite|improve this answer












        share|cite|improve this answer



        share|cite|improve this answer










        answered Dec 11 '18 at 15:51









        BatominovskiBatominovski

        1




        1






























            draft saved

            draft discarded




















































            Thanks for contributing an answer to Mathematics Stack Exchange!


            • Please be sure to answer the question. Provide details and share your research!

            But avoid



            • Asking for help, clarification, or responding to other answers.

            • Making statements based on opinion; back them up with references or personal experience.


            Use MathJax to format equations. MathJax reference.


            To learn more, see our tips on writing great answers.




            draft saved


            draft discarded














            StackExchange.ready(
            function () {
            StackExchange.openid.initPostLogin('.new-post-login', 'https%3a%2f%2fmath.stackexchange.com%2fquestions%2f3035390%2fh-is-a-subgroup-of-a-finite-group-g-such-that-h-and-biggh-1-big%23new-answer', 'question_page');
            }
            );

            Post as a guest















            Required, but never shown





















































            Required, but never shown














            Required, but never shown












            Required, but never shown







            Required, but never shown

































            Required, but never shown














            Required, but never shown












            Required, but never shown







            Required, but never shown







            Popular posts from this blog

            Quarter-circle Tiles

            build a pushdown automaton that recognizes the reverse language of a given pushdown automaton?

            Mont Emei